LSAT and Law School Admissions Forum

Get expert LSAT preparation and law school admissions advice from PowerScore Test Preparation.

User avatar
 Dave Killoran
PowerScore Staff
  • PowerScore Staff
  • Posts: 5852
  • Joined: Mar 25, 2011
|
#88723
Complete Question Explanation
(The complete setup for this game can be found here: lsat/viewtopic.php?f=231&p=88698)

The correct answer choice is (A)

If the number of batches made on Friday is exactly one, then Template #1 is in effect. In that template, Friday is now closed to further batches:

G3-Q17-d1.png

P2 must now be made on Wednesday, and P3 on Thursday. O3 rotates between Wednesday and Thursday, and S1 rotates between Monday, Tuesday, and Wednesday (not shown on the diagram below):

G3-Q17-d2.png

Accordingly, answer choice (A) could be true and is correct.
You do not have the required permissions to view the files attached to this post.

Get the most out of your LSAT Prep Plus subscription.

Analyze and track your performance with our Testing and Analytics Package.